Projection to the subspace spanned by a vector

Johns Hopkins University math exam problems and solutions

Problem 60

Let $T: \R^3 \to \R^3$ be the linear transformation given by orthogonal projection to the line spanned by $\begin{bmatrix}
1 \\
2 \\
2
\end{bmatrix}$.

(a) Find a formula for $T(\mathbf{x})$ for $\mathbf{x}\in \R^3$.

(b) Find a basis for the image subspace of $T$.

(c) Find a basis for the kernel subspace of $T$.

(d) Find the $3 \times 3$ matrix for $T$ with respect to the standard basis for $\R^3$.

(e) Find a basis for the orthogonal complement of the kernel of $T$. (The orthogonal complement is the subspace of all vectors perpendicular to a given subspace, in this case, the kernel.)

(f) Find a basis for the orthogonal complement of the image of $T$.

(g) What is the rank of $T$?

(Johns Hopkins University Exam)

LoadingAdd to solve later

Sponsored Links


Proof.

(a) Find a formula for $T(\mathbf{x})$ for $\mathbf{x}\in \R^3$

For any vector $\mathbf{x}=\begin{bmatrix}
x_1 \\
x_2 \\
x_3
\end{bmatrix}\in \R^3$,
we have $\mathbf{x}=T(\mathbf{x})+\mathbf{v}$, where $\mathbf{v}=\mathbf{x}-T(\mathbf{x})$, which is perpendicular to the vector $\begin{bmatrix}
1 \\
2 \\
2
\end{bmatrix}$.
Since $T(\mathbf{x})\in W$, we have $T(\mathbf{x})=t\begin{bmatrix}
1 \\
2 \\
2
\end{bmatrix}$ for some number $t$.
Thus $\mathbf{x}=t\begin{bmatrix}
1 \\
2 \\
2
\end{bmatrix}+\mathbf{v}$.
To determine the number $t$, we take the inner product with $\begin{bmatrix}
1 \\
2 \\
2
\end{bmatrix}$ and obtain
\begin{align*}
\mathbf{x}\cdot \begin{bmatrix}
1 \\
2 \\
2
\end{bmatrix}
=t\begin{bmatrix}
1 \\
2 \\
2
\end{bmatrix}\cdot \begin{bmatrix}
1 \\
2 \\
2
\end{bmatrix} + \mathbf{v} \cdot \begin{bmatrix}
1 \\
2 \\
2
\end{bmatrix} \\
\Leftrightarrow \,\,\,\, x_1+2x_2+2x_3=9t
\end{align*}
Here $\mathbf{v} \cdot \begin{bmatrix}
1 \\
2 \\
2
\end{bmatrix}=\mathbf{0}$ since $\mathbf{v}$ is perpendicular to $\begin{bmatrix}
1 \\
2 \\
2
\end{bmatrix}$.
Therefore we have $t=\frac{1}{9}(x_1+2x_2+2x_3)$, and the formula is
\[T(\mathbf{x})=\frac{1}{9}(x_1+2x_2+2x_3)\begin{bmatrix}
1 \\
2 \\
2
\end{bmatrix}.\]

 

(b) Find a basis for the image subspace of $T$

Let $V$ be the subspace spanned by $\begin{bmatrix}
1 \\
2 \\
2
\end{bmatrix}$ in $\R^3$.
Since $T$ is a projection to the subspace $W$, the image is $W$ itself.
(Any vector in $W$ is mapped to itself by the projection $T$.) Since $W$ is spanned by just one vector $\begin{bmatrix}
1 \\
2 \\
2
\end{bmatrix}$, it is one-dimensional and a basis is $\left\{\, \begin{bmatrix}
1 \\
2 \\
2
\end{bmatrix} \,\right\}$.

(c) Find a basis for the kernel subspace of $T$

If $\mathbf{x}=\begin{bmatrix}
x_1 \\
x_2 \\
x_3
\end{bmatrix} \in \ker T$, then by the formula in part (a), we have $x_1+2x_2+2x_3=0$.
Thus the vectors in the kernel of $T$ can be written as
\[\mathbf{x}=\begin{bmatrix}
-2s-2t \\
s \\
t
\end{bmatrix}=
\begin{bmatrix}
-2 \\
1 \\
0
\end{bmatrix}s+
\begin{bmatrix}
-2 \\
0 \\
1
\end{bmatrix}t,\] where $s$ and $t$ are free variables.
Since the vectors $\begin{bmatrix}
-2 \\
1 \\
0
\end{bmatrix}$ and $\begin{bmatrix}
-2 \\
0 \\
1
\end{bmatrix}$ are linearly independent and they span the kernel, the basis of $\ker T$ is
\[ \left \{\, \begin{bmatrix}
-2 \\
1 \\
0
\end{bmatrix}, \begin{bmatrix}
-2 \\
0 \\
1
\end{bmatrix}
\,\right \}.\]

(d) Find the $3 \times 3$ matrix for $T$ with respect to the standard basis for $\R^3$

 The matrix for $T$ is given by $[T(\mathbf{e}_1), T(\mathbf{e}_2), T(\mathbf{e}_3)]$, where $\mathbf{e}_1, \mathbf{e}_2, \mathbf{e}_3$ are the standard basis unit vectors for $\R^3$.
By the formula in part (a) we compute
\[T(\mathbf{e}_1)=\frac{1}{9}\begin{bmatrix}
1 \\
2 \\
2
\end{bmatrix}, T(\mathbf{e}_2)=\frac{2}{9}\begin{bmatrix}
1 \\
2 \\
2
\end{bmatrix}, T(\mathbf{e}_3)=\frac{2}{9}\begin{bmatrix}
1 \\
2 \\
2
\end{bmatrix}.\] Therefore the matrix for $T$ with respect to the standard basis is
\[\frac{1}{9}\begin{bmatrix}
1 & 2 & 2 \\
2 &4 &4 \\
2 & 4 & 4
\end{bmatrix}.\]

(e) Find a basis for the orthogonal complement of the kernel of $T$

Note that the kernel consists of vectors in $\R^3$ that are perpendicular to $\begin{bmatrix}
1 \\
2 \\
2
\end{bmatrix}$.
Therefore the vectors perpendicular to the vectors in the kernel is parallel to $\begin{bmatrix}
1 \\
2 \\
2
\end{bmatrix}$.
Thus a basis for the orthogonal complement is $\left\{\, \begin{bmatrix}
1 \\
2 \\
2
\end{bmatrix} \,\right \}$.

(f) Find a basis for the orthogonal complement of the image of $T$

The image of $T$ is the subspace $W$ spanned by $\begin{bmatrix}
1 \\
2 \\
2
\end{bmatrix}$.
Thus the orthogonal complement of the image is the same as the kernel of $T$. Thus a basis is
\[ \left \{\, \begin{bmatrix}
-2 \\
1 \\
0
\end{bmatrix}, \begin{bmatrix}
-2 \\
0 \\
1
\end{bmatrix}
\,\right\}\] as we saw in part (c).

(g) What is the rank of $T$?

The rank of $T$ is the dimension of the image of $T$. The image is $T$ and it is one-dimensional since it is spanned by only one vector. Thus the rank of $T$ is $1$.


LoadingAdd to solve later

Sponsored Links

More from my site

  • Subspace Spanned By Cosine and Sine FunctionsSubspace Spanned By Cosine and Sine Functions Let $\calF[0, 2\pi]$ be the vector space of all real valued functions defined on the interval $[0, 2\pi]$. Define the map $f:\R^2 \to \calF[0, 2\pi]$ by \[\left(\, f\left(\, \begin{bmatrix} \alpha \\ \beta \end{bmatrix} \,\right) \,\right)(x):=\alpha \cos x + \beta […]
  • Determine When the Given Matrix InvertibleDetermine When the Given Matrix Invertible For which choice(s) of the constant $k$ is the following matrix invertible? \[A=\begin{bmatrix} 1 & 1 & 1 \\ 1 &2 &k \\ 1 & 4 & k^2 \end{bmatrix}.\]   (Johns Hopkins University, Linear Algebra Exam)   Hint. An $n\times n$ matrix is […]
  • Determine Eigenvalues, Eigenvectors, Diagonalizable From a Partial Information of a MatrixDetermine Eigenvalues, Eigenvectors, Diagonalizable From a Partial Information of a Matrix Suppose the following information is known about a $3\times 3$ matrix $A$. \[A\begin{bmatrix} 1 \\ 2 \\ 1 \end{bmatrix}=6\begin{bmatrix} 1 \\ 2 \\ 1 \end{bmatrix}, \quad A\begin{bmatrix} 1 \\ -1 \\ 1 […]
  • Idempotent Matrices are DiagonalizableIdempotent Matrices are Diagonalizable Let $A$ be an $n\times n$ idempotent matrix, that is, $A^2=A$. Then prove that $A$ is diagonalizable.   We give three proofs of this problem. The first one proves that $\R^n$ is a direct sum of eigenspaces of $A$, hence $A$ is diagonalizable. The second proof proves […]
  • Find a Linear Transformation Whose Image (Range) is a Given SubspaceFind a Linear Transformation Whose Image (Range) is a Given Subspace Let $V$ be the subspace of $\R^4$ defined by the equation \[x_1-x_2+2x_3+6x_4=0.\] Find a linear transformation $T$ from $\R^3$ to $\R^4$ such that the null space $\calN(T)=\{\mathbf{0}\}$ and the range $\calR(T)=V$. Describe $T$ by its matrix […]
  • Linear Transformation to 1-Dimensional Vector Space and Its KernelLinear Transformation to 1-Dimensional Vector Space and Its Kernel Let $n$ be a positive integer. Let $T:\R^n \to \R$ be a non-zero linear transformation. Prove the followings. (a) The nullity of $T$ is $n-1$. That is, the dimension of the nullspace of $T$ is $n-1$. (b) Let $B=\{\mathbf{v}_1, \cdots, \mathbf{v}_{n-1}\}$ be a basis of the […]
  • A Matrix Representation of a Linear Transformation and Related SubspacesA Matrix Representation of a Linear Transformation and Related Subspaces Let $T:\R^4 \to \R^3$ be a linear transformation defined by \[ T\left (\, \begin{bmatrix} x_1 \\ x_2 \\ x_3 \\ x_4 \end{bmatrix} \,\right) = \begin{bmatrix} x_1+2x_2+3x_3-x_4 \\ 3x_1+5x_2+8x_3-2x_4 \\ x_1+x_2+2x_3 \end{bmatrix}.\] (a) Find a matrix $A$ such that […]
  • Null Space, Nullity, Range, Rank of a Projection Linear TransformationNull Space, Nullity, Range, Rank of a Projection Linear Transformation Let $\mathbf{u}=\begin{bmatrix} 1 \\ 1 \\ 0 \end{bmatrix}$ and $T:\R^3 \to \R^3$ be the linear transformation \[T(\mathbf{x})=\proj_{\mathbf{u}}\mathbf{x}=\left(\, \frac{\mathbf{u}\cdot \mathbf{x}}{\mathbf{u}\cdot \mathbf{u}} \,\right)\mathbf{u}.\] (a) […]

You may also like...

Leave a Reply

Your email address will not be published. Required fields are marked *

This site uses Akismet to reduce spam. Learn how your comment data is processed.

More in Linear Algebra
A Square Root Matrix of a Symmetric Matrix

Answer the following two questions with justification. (a) Does there exist a $2 \times 2$ matrix $A$ with $A^3=O$ but $A^2...

Close